Should I Shift by 1 or 4? Understanding Fourier Transform Time Shift

Click For Summary
The discussion revolves around determining the correct shift for the Fourier Transform of the signal x(t) = sin(4(t-1)). The confusion lies in whether to shift by 1 or 4, with the consensus leaning towards a shift of 4, leading to the multiplication of F{sin(4t)} by e^(4jw). Participants also touch on the notation difference between 'i' and 'j' for imaginary numbers, noting that engineers use 'j' to avoid confusion with current, represented by 'i'. Overall, the main focus is on the correct application of time shifting in Fourier Transforms. Understanding these shifts is crucial for accurate signal analysis.
EvLer
Messages
454
Reaction score
0
I am given this signal:
x(t) = sin(4(t-1))
and I need to find X(jw), i.e. it's FT, so I am confused whether I shift by 1 or by 4, in other words whether I multiply F{sin(4t)} by e^(4jw) or by e^(1jw)

which one is it? I am thinking it's 4jw... is it right?
 
Physics news on Phys.org
Aarg! Those engineers and their jmaginary numbers!

I hope you will bear with me as I use "i" rather than "j". I just can't force myself to that heresy.

sin(4(t-1))= sin(4t- 4). Looks to me like you need to shift by 4.
 
HallsofIvy said:
Aarg! Those engineers and their jmaginary numbers!
:-p :biggrin:

thanks
 
a bit off topic

perhaps i should have started a different thread, but since HallsofIvy has mentioned, i couldn't help but ask: is there any tiniest difference between i and j, i.e. are we, engineers, underestimate imaginary numbers in some way?
It always comes up and i just would like to satisfy my curiosity.:bugeye:
 
Mr EvLer,

There is no big deal in taking j instead of i for imaginary numbers. For engineers, 'i' represents current. To avoid confusion engineers take 'j' for imaginary part.
 
Question: A clock's minute hand has length 4 and its hour hand has length 3. What is the distance between the tips at the moment when it is increasing most rapidly?(Putnam Exam Question) Answer: Making assumption that both the hands moves at constant angular velocities, the answer is ## \sqrt{7} .## But don't you think this assumption is somewhat doubtful and wrong?

Similar threads

Replies
1
Views
2K
  • · Replies 6 ·
Replies
6
Views
2K
  • · Replies 5 ·
Replies
5
Views
2K
  • · Replies 3 ·
Replies
3
Views
2K
Replies
4
Views
2K
  • · Replies 6 ·
Replies
6
Views
3K
  • · Replies 3 ·
Replies
3
Views
2K
  • · Replies 1 ·
Replies
1
Views
2K
  • · Replies 5 ·
Replies
5
Views
3K
Replies
8
Views
2K